LSAT and Law School Admissions Forum

Get expert LSAT preparation and law school admissions advice from PowerScore Test Preparation.

 Administrator
PowerScore Staff
  • PowerScore Staff
  • Posts: 8916
  • Joined: Feb 02, 2011
|
#40563
Complete Question Explanation
(The complete setup for this game can be found here: lsat/viewtopic.php?t=15423)

The correct answer choice is (C)

The question stem creates a Z :longline: Y sequence. When the first rule is added to this new condition, a V :longline: Z :longline: Y sequence results. From the fourth rule, Y must be among the first three slots, and so the only placement for the three variables in the sequence is that Y must perform in slot three, Z must perform in slot two, and V must perform in slot one:
PT70 -Game_#1_#2_diagram 1.png
The last three slots are filled by U, W, and X, with the additional restriction that W X from the second rule. As there is no restriction on W performing in slot four, answer choice (C) is the correct answer.

Get the most out of your LSAT Prep Plus subscription.

Analyze and track your performance with our Testing and Analytics Package.